Difference between revisions of "2021 Fall AMC 12B Problems/Problem 18"
(→Solution) |
m (typo) |
||
Line 30: | Line 30: | ||
Induction: <math>n_{k+1}=2*2^k+2-1=2^{k+1}+1</math> | Induction: <math>n_{k+1}=2*2^k+2-1=2^{k+1}+1</math> | ||
− | It follows that <math>n_{10 | + | It follows that <math>n_{10}=2^{10}+1>1000</math>, and <math>n_9=2^9+1<1000</math>. Therefore, the least value of <math>k</math> would be <math>\boxed{\textbf{(A) }10}</math>. |
+ | |||
+ | -ConcaveTriangle |
Revision as of 17:19, 24 November 2021
Problem
Set , and for let be determined by the recurrence
This sequence tends to a limit; call it . What is the least value of such that
Solution
If we list out the first few values of k, we get the series , which seem to always be a negative power of 2 away from . We can test this out by setting to .
Now,
This means that .
We see that seems to always be above a power of . We can prove this using induction.
Claim:
Base case:
Induction:
It follows that , and . Therefore, the least value of would be .
-ConcaveTriangle